Javascript required
Skip to content Skip to sidebar Skip to footer

Multiple Choice Questions on Limits and Continuity Pdf

Latest Limit and Continuity MCQ Objective Questions

Limit and Continuity MCQ Question 1:

What is \(\lim\limits_{x \to 0}\frac{sinx~+~log(1-x)}{x^2}\)  equal to?

  1. 0
  2. 1
  3. -1/2
  4. 1/2

Answer (Detailed Solution Below)

Option 3 : -1/2

Concept:

L-Hospital Rule: Let f(x) and g(x) be two functions

Suppose that we have one of the following cases,

  1. \(\lim\limits_{x \to a} \frac{f(x)}{g(x)} =\frac{0}{0}\)
  2. \(\lim\limits_{x \to a} \frac{f(x)}{g(x)} =\frac{\infty }{\infty }\)

Then we can apply L-Hospital Rule ⇔\(\lim\limits_{x \to a} \frac{f(x)}{g(x)} =\lim\limits_{x \to a} \frac{f'(x)}{g'(x)}\)

Note: We have to differentiate both the numerator and denominator with respect to x unless and until

\(\lim\limits_{x \to a} \frac{f(x)}{g(x)} =l\neq \frac{0}{0}\) where l is a finite value.

Calculation:

\(\lim\limits_{x \to 0}\frac{sinx~+~log(1-x)}{x^2}\)

Form of limit\(\frac{0}{0}\)

Applying the L-Hospital rule,

\(\Rightarrow \displaystyle \lim_{ x\to 0}\frac{cosx+\frac{1}{1-x}(-1)}{2x}\)

\(\Rightarrow\displaystyle \lim_{ x\to 0}\frac{cosx-\frac{1}{1-x}}{2x}\)

\(\Rightarrow\displaystyle \lim_{ x\to 0}\frac{cosx-({1-x)^{-1}}}{2x}\)

Now Again Form of limit\(\frac{0}{0}\)

Applying the L-Hospital rule,

\(\Rightarrow\displaystyle \lim_{ x\to 0}\frac{-sinx-\frac{(1-x)-(-1)}{(1-x)^{2}}}{2}\)

\(\Rightarrow\displaystyle \lim_{ x\to 0}\frac{-sin~x-[(-1)(1-x)^{-2}(-1)]}{2}\)

\(\Rightarrow\displaystyle \lim_{ x\to 0}\frac{-sin~x-[(1-x)^{-2}]}{2}\)

\(\Rightarrow\displaystyle \lim_{ x\to 0}\frac{-sin~0-[(1-0)^{-2}]}{2}\)

=\(\frac{0-1}{2}\)

= -1/2

Limit and Continuity MCQ Question 2:

The value of k which makes \(f\left( x \right)\; = \;\left\{ {\begin{array}{*{20}{c}} {\frac{sin~5x}{x}\;,x \ne 0}\\ {k\;,x\; = \;0} \end{array}} \right.\)  continuous at x = 0, is

  1. 2
  2. 3
  3. 4
  4. 5

Answer (Detailed Solution Below)

Option 4 : 5

Concept:

For Continuity of the function f(x), Left hand limit = Right hand limit

i.e.\(\displaystyle \lim_{ x\to 0^{-}}\)f(x) =\(\displaystyle \lim_{ x\to 0^{+}}\)f(x) =\(\displaystyle \lim_{ x\to 0^{}}f(x)\)

\(\displaystyle \lim_{ x\to 0^{}}\frac{sinx}{x}=1\).........(Equation 1)

Application:

For Continuity of the function f(x), Left hand limit = Right hand limit

i.e.\(\displaystyle \lim_{ x\to 0^{-}}\)f(x) =\(\displaystyle \lim_{ x\to 0^{+}}\)f(x) =\(\displaystyle \lim_{ x\to 0^{}}f(x)\)

\(\displaystyle \lim_{ h\to 0^{}}\frac{sin5 (0-h)}{0-h}=\displaystyle \lim_{ h\to 0^{}}\frac{sin5 (0+h)}{0-+h}\)=\(\displaystyle \lim_{ x\to 0^{}} f(x)\)

\(\displaystyle \lim_{ h\to 0^{}}\frac{sin5 (0-h)}{0-h} \times \frac{5}{5}=\displaystyle \lim_{ h\to 0^{}}\frac{sin5 (0+h)}{0+h}\times\frac{5}{5}\) = k

\(5\displaystyle \lim_{ h\to 0^{}}\frac{sin5h}{5h}\) =\(5\displaystyle \lim_{ h\to 0^{}}\frac{sin5h}{5h}\) = k (Using equation 1)

5(1) = k

\(\Rightarrow\)k=5

Limit and Continuity MCQ Question 3:

What is \(\lim\limits_{x \to 1}\frac{2(1-cosx)}{x^2}\)  equal to?

  1. \(4sin^2\frac{1}{3}\)
  2. \(4sin^2\frac{1}{2}\)
  3. \(sin^2\frac{1}{2}\)
  4. \(0\)

Answer (Detailed Solution Below)

Option 2 : \(4sin^2\frac{1}{2}\)

Concept:

1-cosx=2\(sin^2\frac{x}{2}\)

Calculation:

In the given question, We have\(\displaystyle \lim_{ x \to 1} \frac{2(1-cosx)}{x^2}\)i.e.

\(\Rightarrow\) \(\displaystyle \lim_{ x \to 1} \frac{2(1-cosx)}{x^2}\)=\(\displaystyle \lim_{ x\to 1} \frac{(2)2sin^2\frac{x}{2}}{x^2}\)

\(\Rightarrow\) \(\displaystyle \lim_{ x \to 1} \frac{2(1-cosx)}{x^2} =\) \(\displaystyle \lim_{x \to 1} \frac {sin^2\frac{x}{2}}{\frac{x^2}{2^2}}\)

\(\Rightarrow\) \(\displaystyle \lim_{ x \to 1} \frac{2(1-cosx)}{x^2} =\) \(\frac{sin^2\frac{1}{2}}{\frac{1^2}{2^2}}\)

\(\Rightarrow\) \(\displaystyle \lim_{ x \to 1} \frac{2(1-cosx)}{x^2} =\) \(4sin^2\frac{1}{2}\)

Limit and Continuity MCQ Question 4:

The point(s), at which the function f given by f(x) = \(\left\{ \begin{array}{l} \frac{x}{{|x|}},x < 0\\ - 1,x \ge 0 \end{array} \right.\)  is continuous, is/are :

  1. x ∈ R
  2. x = 0
  3. x ∈ R - {0}
  4. x = -1 and 1

Answer (Detailed Solution Below)

Option 1 : x ∈ R

Concept:

A function f(x) is continuous at x = a, if\(\lim_{x\to a^{-}}\) f(x) =\(\lim_{x\to a^{+}}\)f(x) = f(a).

Explanation:

Given, f(x) =\(\left\{ \begin{array}{l} \frac{x}{{|x|}},x < 0\\ - 1,x \ge 0 \end{array} \right.\)

 We know that |x| = x when x > 0

and |x | = - x when x < 0

f(x) =\(\left\{ \begin{array}{l} \frac{x}{{(-x)}},x < 0\\ - 1,x \ge 0 \end{array} \right.\)

⇒ f(x) =\(\left\{ \begin{array}{l} -1,x < 0\\ - 1,x \ge 0 \end{array} \right.\)

Therefore, f(x) = -1

The function f(x) is a constant function, and a constant function is continuous for all x ∈ R.

The correct answer is option (1).

Limit and Continuity MCQ Question 5:

What is \(\displaystyle\lim_{h \rightarrow 0} \frac{\sin^2(x+h)−\sin^2x}{h}\)  equal to ?

  1. sin 2  x
  2. cos 2  x
  3. sin 2x
  4. cos 2x

Answer (Detailed Solution Below)

Option 3 : sin 2x

Concept:

L-Hospital Rule: Let f(x) and g(x) be two functions

Suppose that we have one of the following cases,

I.\(\lim\limits_{x \to a} \frac{f(x)}{g(x)} =\frac{0}{0}\)

II.\(\lim\limits_{x \to a} \frac{f(x)}{g(x)} =\frac{\infty }{\infty }\)

Then we can apply L-Hospital Rule ⇔\(\lim\limits_{x \to a} \frac{f(x)}{g(x)} =\lim\limits_{x \to a} \frac{f'(x)}{g'(x)}\)

Note: We have to differentiate both the numerator and denominator with respect to x unless and until

\(\lim\limits_{x \to a} \frac{f(x)}{g(x)} =l\neq \frac{0}{0}\) where l is a finite value.

Solution:

\(\displaystyle\lim_{h \rightarrow 0} \frac{\sin^2(x+h)−\sin^2x}{h}\)

Form of limit\(\frac{0}{0}\)

Applying the L-Hospital rule,

=\(\displaystyle \lim_{h \to 0}\frac{2sin(x+h)cos(x+h)-0}{1}\)

= 2sin(x + 0)cos(x + 0)

= 2sinx.cosx

= sin2x

∴ The correct option is (3)

Top Limit and Continuity MCQ Objective Questions

Find the value of\(\rm \displaystyle \lim_{x \rightarrow \infty} 2x \sin \left(\frac{4} {x}\right)\)

  1. 2
  2. 4
  3. 8
  4. \(\frac 1 2\)

Answer (Detailed Solution Below)

Option 3 : 8

Concept:

\(\rm \displaystyle \lim_{x \rightarrow 0} \frac{\sin x}{x} = 1\)

Calculation:

\(\rm \displaystyle \lim_{x \rightarrow \infty} 2x \sin \left(\frac{4} {x}\right)\)

=\(\rm 2 \times \displaystyle \lim_{x → ∞} \frac{\sin \left(\frac{4} {x}\right)}{\left(\frac{1}{x} \right )}\)

=\(\rm 2 \times \displaystyle \lim_{x → ∞} \frac{\sin \left(\frac{4} {x}\right)}{\left(\frac{4}{x} \right )} \times 4\)

Let\(\rm \frac {4}{x} = t\)

If x → ∞ then t → 0

=\(\rm 8 \times\displaystyle \lim_{t \rightarrow 0} \frac{\sin t}{t} \)

= 8 × 1

= 8

What is the value of\(\mathop {{\rm{lim}}}\limits_{x \to 0} \;\;\frac{{{{\left( {1 - \cos2x} \right)}^2}\;}}{{{x^4}}}\)

  1. 1
  2. 8
  3. 4
  4. 0

Answer (Detailed Solution Below)

Option 3 : 4

Concept:

  • 1 - cos 2θ = 2 sin2 θ
  • \(\mathop {{\rm{lim}}}\limits_{x \to 0} \;\;\frac{{\sin x}}{x} = 1\)

Calculation:

\(\mathop {{\rm{lim}}}\limits_{x \to 0} \;\;\frac{{{{\left( {1 - \cos2x} \right)}^2}\;}}{{{x^4}}}\)

=\(\;\mathop {{\rm{lim}}}\limits_{x \to 0} \;\;\frac{{{{\left( {2{{\sin }^2}x} \right)}^2}}}{{{x^4}}}\)          (1 - cos 2θ = 2 sin2 θ)

=\(\;\mathop {{\rm{lim}}}\limits_{x \to 0} \;\;\frac{{4{{\sin }^4}x}}{{{x^4}}}\)

=\(\mathop {\lim }\limits_{x \to 0} 4\; × \;{\left( {\frac{{\sin x}}{x}} \right)^4}\)

= 4 × 1 = 4

Evaluate\(\rm \mathop {\lim }\limits_{x\rightarrow \infty} \frac{x}{\sqrt{1+2x^2}}\)

  1. 0
  2. 1
  3. \(\frac{1}{\sqrt 2}\)
  4. \(\frac 1 2\)

Answer (Detailed Solution Below)

Option 3 : \(\frac{1}{\sqrt 2}\)

Calculation:

We have to find the value of\(\rm \mathop {\lim }\limits_{x\rightarrow ∞} \frac{x}{\sqrt{1+2x^2}}\)

\(\rm \mathop {\lim }\limits_{x\rightarrow ∞} \frac{x}{\sqrt{1+2x^2}}\)       [Form\(\frac{∞}{∞}\)]

This limit is of the form\(\frac{∞}{∞}\), Here, We can cancel a factor going to ∞  out of the numerator and denominator.

\(\rm \mathop {\lim }\limits_{x\rightarrow ∞} \frac{x}{\sqrt{1+2x^2}}\)

=\(\rm \mathop {\lim }\limits_{x\rightarrow \infty} \frac{x}{x\sqrt{\frac{1}{x^2}+2}}\)

Factor x becomes ∞ at x tends to ∞, So we need to cancel this factor from numerator and denominator.

=\(\rm \mathop {\lim }\limits_{x\rightarrow \infty} \frac{1}{\sqrt{\frac{1}{x^2}+2}}\)

=\(\frac{1}{\sqrt{\frac{1}{\infty^2}+2}}=\frac{1}{\sqrt{0+2}}=\frac{1}{\sqrt 2}\)

Evaluate\(\rm \mathop {\lim }\limits_{x\rightarrow 0} \frac{\log (1+2x)}{\tan 2x}\)

  1. -1
  2. 1
  3. 2
  4. 4

Answer (Detailed Solution Below)

Option 2 : 1

Concept:

\(\rm \mathop {\lim }\limits_{x\; \to \;a} \left[ {\frac{{f\left( x \right)}}{{g\left( x \right)}}} \right] = \frac{{\mathop {\lim }\limits_{x\; \to \;a} f\left( x \right)}}{{\mathop {\lim }\limits_{x\; \to \;a} g\left( x \right)}},\;provided\;\mathop {\lim }\limits_{x\; \to a} g\left( x \right) \ne 0\)

\(\rm \mathop {\lim }\limits_{x\; \to 0} {\frac{{\tan x}}{x}} = 1\)

\(\rm \mathop {\lim }\limits_{x\; \to 0} {\frac{{\log (1+x)}}{x}} = 1\)

Calculation:

\(\rm \mathop {\lim }\limits_{x\rightarrow 0} \frac{\log (1+2x)}{\tan 2x}\)

\(\rm = \mathop {\lim }\limits_{x\rightarrow 0} \frac{\frac{\log (1+2x)}{2x} \times 2x}{\frac{\tan 2x}{2x} \times 2x}\\= \frac{\mathop {\lim }\limits_{x\rightarrow 0}\frac{\log (1+2x)}{2x} }{\mathop {\lim }\limits_{x\rightarrow 0}\frac{\tan 2x}{2x} }\)

As we know\(\rm \mathop {\lim }\limits_{x\; \to 0} {\frac{{\tan x}}{x}} = 1\) and\(\rm \mathop {\lim }\limits_{x\; \to 0} {\frac{{\log (1+x)}}{x}} = 1\)

Therefore,\(\rm \mathop {\lim }\limits_{x\; \to 0} {\frac{{\tan 2x}}{2x}} = 1\) and\(\rm \mathop {\lim }\limits_{x\; \to 0} {\frac{{\log (1+2x)}}{2x}} = 1\)

Hence\(\rm \mathop {\lim }\limits_{x\rightarrow 0} \frac{\log (1+2x)}{\tan 2x} = \frac 1 1=1\)

Evaluate\(\rm \mathop {\lim }\limits_{x\rightarrow \infty} \frac{x^2}{{1+x^2}}\)

  1. 0
  2. 1
  3. 2
  4. \(\frac 12 \)

Answer (Detailed Solution Below)

Option 2 : 1

Calculation:

We have to find the value of\(\rm \mathop {\lim }\limits_{x\rightarrow \infty} \frac{x^2}{{1+x^2}}\)

\(\rm \mathop {\lim }\limits_{x\rightarrow \infty} \frac{x^2}{{1+x^2}}\)       [Form\(\frac{∞}{∞}\)]

This limit is of the form\(\frac{∞}{∞}\), Here, We can cancel a factor going to ∞  out of the numerator and denominator.

\(\rm \mathop {\lim }\limits_{x\rightarrow \infty} \frac{x^2}{{1+x^2}}\)

=\(\rm \mathop {\lim }\limits_{x\rightarrow \infty} \frac{x^2}{x^2\left({\frac {1}{x^2}+1}\right)}\)

Factor x2 becomes ∞ at x tends to ∞, So we need to cancel this factor from numerator and denominator.

=\(\rm \mathop {\lim }\limits_{x\rightarrow \infty} \frac{1}{\left({\frac {1}{x^2}+1}\right)}\)

=\(\frac{1}{{\frac{1}{\infty^2}+1}}=\frac{1}{{0+1}}=1\)

Examine the continuity of a function f(x) = (x - 2) (x - 3)

  1. Discontinuous at x = 2
  2. Discontinuous at x = 2, 3
  3. Continuous everywhere
  4. Discontinuous at x = 3

Answer (Detailed Solution Below)

Option 3 : Continuous everywhere

Concept:

  • We say f(x) is continuous at x = c if

LHL = RHL = value of f(c)

i.e.,\(\mathop {\lim }\limits_{{\rm{x}} \to {{\rm{c}}^ - }} {\rm{f}}\left( {\rm{x}} \right) = \mathop {\lim }\limits_{{\rm{x}} \to {{\rm{c}}^ + }} {\rm{f}}\left( {\rm{x}} \right) = {\rm{f}}\left( {\rm{c}} \right)\)

Calculation:

\(\mathop {\lim }\limits_{{\rm{x}} \to {\rm{a}}} {\rm{f}}\left( {\rm{x}} \right)\; = \;\mathop {{\rm{lim}}}\limits_{{\rm{x}} \to {\rm{a}}} \left( {{\rm{x}} - 2} \right)\left( {{\rm{x}} - 3} \right)\)            (a ϵ Real numbers)

\(= \mathop {{\rm{lim}}}\limits_{{\rm{x}} \to {\rm{a}}} {\rm{\;}}{{\rm{x}}^2} - 3{\rm{x}} - 2{\rm{x}} + 6\)

\(= \mathop {{\rm{lim}}}\limits_{{\rm{x}} \to {\rm{a}}} {\rm{\;}}{{\rm{x}}^2} - 5{\rm{x}} + 6\)

\(= {{\rm{a}}^2} - 5{\rm{a}} + 6\)

∴ f(x) = f(a), So continuous at everywhere

Important tip:

Quadratic and polynomial functions are continuous at each point in their domain

\(\mathop {\lim }\limits_{{\rm{x}} \to 0} \frac{{\sin \left( {{{\rm{x}}^3}} \right)}}{{{{\rm{x}}^2}{\rm{\;sinx}}}}\) is equal to

  1. 0
  2. 1
  3. Can't say

Answer (Detailed Solution Below)

Option 3 : 1

Concept:

  • \(\mathop {\lim }\limits_{{\rm{\alpha }} \to 0} \frac{{\sin \left( {\rm{\alpha }} \right)}}{{\rm{\alpha }}} = 1\) and \(\mathop {\lim }\limits_{{\rm{\alpha }} \to 0} \frac{{\rm{\alpha }}}{{\sin \left( {\rm{\alpha }} \right)}} = 1\)
  • \(\mathop {\lim }\limits_{{\rm{x}} \to {\rm{a}}} \left[ {{\rm{f}}\left( {\rm{x}} \right){\rm{\;}} \pm {\rm{\;g}}\left( {\rm{x}} \right)} \right] = {\rm{\;}}\mathop {\lim }\limits_{{\rm{x}} \to {\rm{a}}} {\rm{f}}\left( {\rm{x}} \right){\rm{\;}} \pm \mathop {{\rm{lim}}}\limits_{{\rm{x}} \to {\rm{a}}} {\rm{\;g}}\left( {\rm{x}} \right)\)
  • \(\mathop {\lim }\limits_{{\rm{x}} \to {\rm{a}}} \left[ {\frac{{{\rm{f}}\left( {\rm{x}} \right)}}{{{\rm{g}}\left( {\rm{x}} \right)}}} \right]{\rm{\;}} = \frac{{\mathop {{\rm{lim}}}\limits_{{\rm{x}} \to {\rm{a}}} {\rm{\;f}}\left( {\rm{x}} \right)}}{{\mathop {{\rm{lim}}}\limits_{{\rm{x}} \to {\rm{a}}} {\rm{\;g}}\left( {\rm{x}} \right)}}\)

Calculation:

To find: \(\mathop {\lim }\limits_{{\rm{x}} \to 0} \frac{{\sin \left( {{{\rm{x}}^3}} \right)}}{{{{\rm{x}}^2}{\rm{\;sinx}}}} = ?\)

Multiply and divide by x,

\(\mathop {\lim }\limits_{{\rm{x}} \to 0} \frac{{{\rm{x}}\sin \left( {{{\rm{x}}^3}} \right)}}{{{{\rm{x}}^3}{\rm{\;sinx}}}} = \mathop {\lim }\limits_{{\rm{x}} \to 0} \frac{{\sin \left( {{{\rm{x}}^3}} \right)}}{{{{\rm{x}}^3}}}\frac{{\rm{x}}}{{{\rm{sinx}}}}\)

\(= \mathop {\lim }\limits_{{\rm{x}} \to 0} \frac{{\sin \left( {{{\rm{x}}^3}} \right)}}{{{{\rm{x}}^3}}} \cdot \mathop {{\rm{lim}}}\limits_{{\rm{x}} \to 0} \frac{{\rm{x}}}{{{\rm{sinx}}}}\)

\(= 1 \cdot 1 = 1\)

Hence, option (3) is correct.

If\(\rm f(x)=\left\{\begin{matrix}\rm \dfrac{\sin 3x}{e^{2x}-1}, &\rm x \ne 0\\\rm k-2, &\rm x=0 \end{matrix}\right.\) is continuous at x = 0, then k = ?

  1. \(\dfrac{3}{2}\)
  2. \(\dfrac{9}{5}\)
  3. \(\dfrac{1}{2}\)
  4. \(\dfrac{7}{2}\)

Answer (Detailed Solution Below)

Option 4 : \(\dfrac{7}{2}\)

Concept:

Definition:

  • A function f(x) is said to be continuous at a point x = a in its domain, if\(\rm \displaystyle \lim_{x\to a}f(x)\) exists oror if its graph is a single unbroken curve at that point.
  • f(x) is continuous at x = a ⇔\(\rm \displaystyle \lim_{x\to a^+}f(x)=\lim_{x\to a^-}f(x)=\lim_{x\to a}f(x)=f(a)\).

Formulae:

  • \(\rm \displaystyle \lim_{x\to 0}\dfrac{\sin x}{x}=1\)
  • \(\rm \displaystyle \lim_{x\to 0}\dfrac{e^x-1}{x}=1\)

Calculation:

Since f(x) is given to be continuous at x = 0,\(\rm \displaystyle \lim_{x\to 0}f(x)=f(0)\).

Also,\(\rm \displaystyle \lim_{x\to a^+}f(x)=\lim_{x\to a^-}f(x)\) because f(x) is same for x > 0 and x < 0.

\(\rm \therefore \displaystyle \lim_{x\to 0}f(x)=f(0)\)

\(\rm \Rightarrow \displaystyle \lim_{x\to 0}\dfrac{\sin 3x}{e^{2x}-1}=k-2\)

\(\rm \Rightarrow \displaystyle \lim_{x\to 0}\dfrac{\dfrac{\sin 3x}{3x}\times3x}{\dfrac{e^{2x}-1}{2x}\times2x}=k-2\)

\(\rm \Rightarrow \dfrac{3}{2}=k-2\)

\(\rm \Rightarrow k=\dfrac{7}{2}\).

\(\mathop {\lim }\limits_{n \to \infty } \frac{{{2^{n + 1}} + {3^{n + 1}}}}{{{2^n} + {3^n}}}\) equals

  1. 3
  2. 2
  3. 1
  4. 0

Answer (Detailed Solution Below)

Option 1 : 3

\(\mathop {\lim }\limits_{n \to \infty } \frac{{{2^{n + 1}} + {3^{n + 1}}}}{{{2^n} + {3^n}}} \)

This can be written as:

\(= \mathop {\lim }\limits_{n \to \infty } \frac{{{2^n}2 + {3^n}3}}{{{2^n} + {3^n}}}\)

Taking 3n common, we can write:

\( = \mathop {\lim }\limits_{n \to \infty } \frac{{{3^n}\left[ {2.{{\left( {\frac{2}{3}} \right)}^n} + 3} \right]}}{{{3^n}\left[ {{{\left( {\frac{2}{3}} \right)}^n} + 1} \right]}}\)

\( = \mathop {\lim }\limits_{n \to \infty } \frac{{2.{{\left( {\frac{2}{3}} \right)}^n} + 3}}{{\left[ {{{\left( {\frac{2}{3}} \right)}^n} + 1} \right]}}\)

Here\(\frac 2 3 < 1\)

So,\(\left[ \frac {2}{3}\right]^{\infty} = 0\)

\(= \frac{{0 + 3}}{{0 + 1}} = 3\)

What is\(\rm \displaystyle\lim_{x \rightarrow 0} \dfrac{3^x + 3^{-x}-2}{x}\) equal to ?

  1. 0
  2. -1
  3. 1
  4. Limit does not exist

Answer (Detailed Solution Below)

Option 1 : 0

Concept:

\(\rm ​​\mathop {\lim }\limits_{x\; \to \;a} \left[ {f\left( x \right) + g\left( x \right)} \right] = \;\mathop {\lim }\limits_{x\; \to \;a} f\left( x \right) + \;\mathop {\lim }\limits_{x\; \to \;a} g\left( x \right)\)

\(\rm ​​\mathop {\lim }\limits_{x\; \to \;0} \dfrac { (a^x - 1) }{x} = \log a\)

log mn = n log m

Calculation:

\(\rm \displaystyle\lim_{x \rightarrow 0} \dfrac{3^x + 3^{-x}-2}{x}\\= \displaystyle\lim_{x \rightarrow 0} \dfrac{3^x -1+ 3^{-x}-1}{x}\\= \displaystyle\lim_{x \rightarrow 0} \dfrac{3^x -1}{x}+\displaystyle\lim_{x \rightarrow 0} \dfrac{3^{-x} -1}{x}\\= \displaystyle\lim_{x \rightarrow 0} \dfrac{3^x -1}{x}+\displaystyle\lim_{x \rightarrow 0} \dfrac{(3^{-1})^x -1}{x}\\= \log 3 + \log (3^{-1})\\= \log 3 - \log 3\\=0\)

If\(\rm f(x) = \left\{ \begin{matrix} \rm \frac{2x-\sin^{-1} x}{2x + \tan^{-1} x}; & \rm x \ne 0 \\ \rm K; & \rm x = 0 \end{matrix}\right.\) is a continuous function at x = 0, then the value of k is:

  1. 2
  2. \(\dfrac12\)
  3. 1
  4. None of these

Answer (Detailed Solution Below)

Option 4 : None of these

Concept:

Definition:

  • A function f(x) is said tobe continuous at a point x = a in its domain, if\(\rm \displaystyle \lim_{x\to a}f(x)\) existsoror if its graph is a single unbroken curve at that point.
  • f(x) is continuous at x = a ⇔\(\rm \displaystyle \lim_{x\to a^+}f(x)=\lim_{x\to a^-}f(x)=\lim_{x\to a}f(x)=f(a)\).


Calculation:

For x ≠ 0, the given function can be re-written as:

\(\rm f(x) = \left\{ \begin{matrix} \rm \frac{2x-\sin^{-1} x}{2x + \tan^{-1} x}; & \rm x \ne 0 \\ \rm K; & \rm x = 0 \end{matrix}\right.\)

Since the equation of the function is same for x < 0 and x > 0, we have:

\(\rm \displaystyle \lim_{x\to 0^+}f(x)=\lim_{x\to 0^-}f(x)=\lim_{x\to 0}\frac{2x-\sin^{-1} x}{2x + \tan^{-1} x}\)

=\(\rm \displaystyle \lim_{x\to 0}\frac{2- \frac {\sin^{-1} x} x}{2 +\frac {\tan^{-1} x} x} = \frac {2-1}{2+1} = \frac 1 3\)

For the function to be continuous at x = 0, we must have:

\(\rm \displaystyle \lim_{x\to 0}f(x)=f(0)\)

⇒ K =\(\dfrac{1}{3}\).

The value of\(\rm \mathop {\lim }\limits_{x \to 0} \frac{{x}}{\cos x }\) is

  1. 1
  2. 0
  3. -1

Answer (Detailed Solution Below)

Option 2 : 0

Calculation:

Here we have to find the value of\(\rm \mathop {\lim }\limits_{x \to 0} \frac{{x}}{\cos x }\)

Let L =\(\rm \mathop {\lim }\limits_{x \to 0} \frac{{x}}{\cos x }\)

Now put the value of limit, we get

=\(\rm \frac {0}{\cos 0} = \frac 01=0\)

What is\(\rm \displaystyle\lim_{x\rightarrow 0} \dfrac{\sin x \log (1-x)}{x^2}\) equal to?

  1. -1
  2. Zero
  3. -e
  4. \(\rm -\dfrac{1}{e}\)

Answer (Detailed Solution Below)

Option 1 : -1

Concept:

\(\rm ​​\mathop {\lim }\limits_{x\; \to \;a} \left[ {f\left( x \right) \cdot g\left( x \right)} \right] = \;\mathop {\lim }\limits_{x\; \to \;a} f\left( x \right) \cdot \;\mathop {\lim }\limits_{x\; \to \;a} g\left( x \right)\)

\(\rm ​​\mathop {\lim }\limits_{x\; \to \;0} \dfrac {\sin x }{x} = 1\\\rm ​​\mathop {\lim }\limits_{x\; \to \;0} \dfrac {\log (1+x) }{x}\)

Calculation:

We have to find the value of\(\rm \displaystyle\lim_{x\rightarrow 0} \dfrac{\sin x \log (1-x)}{x^2}\)

As we know,\(\rm ​​\mathop {\lim }\limits_{x\; \to \;a} \left[ {f\left( x \right) \cdot g\left( x \right)} \right] = \;\mathop {\lim }\limits_{x\; \to \;a} f\left( x \right) \cdot \;\mathop {\lim }\limits_{x\; \to \;a} g\left( x \right)\)

\(\therefore \rm \displaystyle\lim_{x\rightarrow 0} \dfrac{\sin x \log (1-x)}{x^2}= \rm ​​\mathop {\lim }\limits_{x\; \to \;0} \dfrac {\sin x }{x} × ​​\mathop {\lim }\limits_{x\; \to \;0} \dfrac {\log (1-x) }{x}\)

= 1 × \(\rm ​​\mathop {\lim }\limits_{x\; \to \;0} \dfrac {\log (1+(-x)) }{x}\)

= \(\rm ​​\mathop {\lim }\limits_{x\; \to \;0} \dfrac {\log (1+(-x)) }{-(-x)}\)

=\(-1 × \rm ​​\mathop {\lim }\limits_{x\; \to \;0} \dfrac {\log (1+(-x)) }{(-x)}\)

= -1 × 1

= -1

Let f(2) = 2 and f'(2) = 2. Then \(\mathop {\lim }\limits_{{\rm{x}} \to 2} \frac{{{\rm{xf}}\left( 2 \right) - 2{\rm{f}}\left( {\rm{x}} \right)}}{{{\rm{x}} - 2}}\) is given by

  1. -4
  2. 2
  3. -2
  4. 4

Answer (Detailed Solution Below)

Option 3 : -2

Concept:

  • L' hospital's Rule: For (0/0), (∞ /∞) form of limits

If suppose,\(\mathop {\lim }\limits_{{\rm{x}} \to {\rm{a}}} \frac{{{\rm{f}}\left( {\rm{x}} \right)}}{{{\rm{g}}\left( {\rm{x}} \right)}} = \frac{0}{0}{\rm{\;OR\;}}\mathop {\lim }\limits_{{\rm{x}} \to {\rm{a}}} \frac{{{\rm{f}}\left( {\rm{x}} \right)}}{{{\rm{g}}\left( {\rm{x}} \right)}} = \frac{{ \pm \infty }}{{ \pm \infty }}{\rm{\;\;}}\)

Then we apply L-Hospital's Rule,

\(\mathop {\lim }\limits_{{\rm{x}} \to {\rm{a}}} \frac{{{\rm{f}}\left( {\rm{x}} \right)}}{{{\rm{g}}\left( {\rm{x}} \right)}} = \mathop {\lim }\limits_{{\rm{x}} \to {\rm{a}}} \frac{{\frac{{\rm{d}}}{{{\rm{dx}}}}\left( {{\rm{f}}\left( {\rm{x}} \right)} \right)}}{{\frac{{\rm{d}}}{{{\rm{dx}}}}\left( {{\rm{g}}\left( {\rm{x}} \right)} \right)}} = \mathop {{\rm{lim}}}\limits_{{\rm{x}} \to {\rm{a}}} \frac{{{\rm{f'}}\left( {\rm{x}} \right)}}{{{\rm{g'}}\left( {\rm{x}} \right)}}{\rm{\;}}\)

Differentiate the numerator and differentiate the denominator and then take limit

Calculation:

Given: f(2) = 2 and f'(2) = 2.

To find: \(\mathop {\lim }\limits_{{\rm{x}} \to 2} \frac{{{\rm{xf}}\left( 2 \right) - 2{\rm{f}}\left( {\rm{x}} \right)}}{{{\rm{x}} - 2}}{\rm{\;}}\)=?

Check the form by putting x = 2

\(\mathop {\lim }\limits_{{\rm{x}} \to 2} \frac{{{\rm{xf}}\left( 2 \right) - 2{\rm{f}}\left( {\rm{x}} \right)}}{{{\rm{x}} - 2}} = \frac{{2\left( 2 \right) - 2\left( 2 \right)}}{{2 - 2}} = \frac{0}{0}\)

Apply L' hospital's rule,

\(\mathop {\lim }\limits_{{\rm{x}} \to 2} \frac{{{\rm{xf}}\left( 2 \right) - 2{\rm{f}}\left( {\rm{x}} \right)}}{{{\rm{x}} - 2}} = \mathop {\lim }\limits_{{\rm{x}} \to 2} \frac{{\frac{{\rm{d}}}{{{\rm{dx}}}}\left( {{\rm{xf}}\left( 2 \right) - 2{\rm{f}}\left( {\rm{x}} \right)} \right)}}{{\frac{{\rm{d}}}{{{\rm{dx}}}}\left( {{\rm{x}} - 2} \right)}} = \mathop {{\rm{lim}}}\limits_{{\rm{x}} \to 2} \frac{{{\rm{f}}\left( 2 \right) - 2{\rm{f'}}\left( {\rm{x}} \right)}}{1}\)

Differentiate the numerator and differentiate the denominator, we get

Now, take limit at x = 2

\(\mathop {{\rm{lim}}}\limits_{{\rm{x}} \to 2} = {\rm{f}}\left( 2 \right) - 2{\rm{f'}}\left( 2 \right)\)

\(= 2 - 2\left( 2 \right) = - 2\)

Option (3) is correct.

What is the value of\(\rm \mathop {{\rm{lim}}}\limits_{x \to 0} \;\;\frac{{{{\left( {1 - \cos2x} \right)}^3}\;}}{{{x^6}}}\)

  1. 2
  2. 4
  3. 8
  4. Limit doesn't exist

Answer (Detailed Solution Below)

Option 3 : 8

Concept:

Formulas:

1 - cos 2θ = 2 sin2 θ

1 + cos 2θ = 2 cos2 θ

\(\rm \mathop {{\rm{lim}}}\limits_{x \to 0} \;\;\frac{{\sin x}}{x} = 1\)

Calculation:

To Find: Value of\(\rm \mathop {{\rm{lim}}}\limits_{x \to 0} \;\;\frac{{{{\left( {1 - \cos2x} \right)}^3}\;}}{{{x^6}}}\)

\(\rm \mathop {{\rm{lim}}}\limits_{x \to 0} \;\;\frac{{{{\left( {1 - \cos2x} \right)}^3}\;}}{{{x^6}}}\)

=\(\rm \mathop {{\rm{lim}}}\limits_{x \to 0} \;\;\frac{{{{\left( {2{{\sin }^2}x} \right)}^3}}}{{{x^6}}}\)          (∵1 - cos 2θ = 2 sin2 θ)

=\(\rm \mathop {{\rm{lim}}}\limits_{x \to 0} \;\;\frac{{8{{\sin }^6}x}}{{{x^6}}}\)

=\(\rm \mathop {\lim }\limits_{x \to 0} 8\; \times \;{\left( {\frac{{\sin x}}{x}} \right)^6}\)

= 8 × 1

= 8

Evaluate\(\rm \displaystyle \lim_{x \rightarrow 0} \frac{1 - \cos 2x }{x^2} \)

  1. 0
  2. 1
  3. 2
  4. does not exists

Answer (Detailed Solution Below)

Option 3 : 2

Concept:

\(\rm \displaystyle \lim_{x \rightarrow 0} \frac{\sin x }{x} =1\)

Trigonometry formulas:

1 - cos 2x = 2sin2 x

1 + cos 2x = 2cos2 x

Calculation:

Here, we have to find the value of the limit\(\rm \displaystyle \lim_{x \rightarrow 0} \frac{1 - \cos 2x }{x^2} \)

As we know, 1 - cos 2x = 2sin2 x

\(\rm \displaystyle \lim_{x \rightarrow 0} \frac{1 - \cos 2x }{x^2} \)

=\(\rm \displaystyle \lim_{x \rightarrow 0} \frac{2\sin^2 x }{x^2} \)

= 2 ×\(\rm \displaystyle \lim_{x \rightarrow 0} \frac{\sin x }{x} × \lim_{x \rightarrow 0} \frac{\sin x }{x}\)

= 2 × 1 × 1

= 2

Function f(x) = tan(2x) will be discontinuous at x =

  1. (2n + 1) π
  2. (n + 1) π/2
  3. (2n + 1) π/2
  4. (2n + 1) π/4

Answer (Detailed Solution Below)

Option 4 : (2n + 1) π/4

Concept:

  • f(c) must be defined. You can't have a hole in the function (denominator is zero i.e. infinity)
  • cos x = 0, where x = odd multiple of π/2 = (2n + 1) π/2

Calculation:

Here, \(\tan \left( {2{\rm{x}}} \right) = \frac{{\sin \left( {2{\rm{x}}} \right)}}{{\cos \left( {2{\rm{x}}} \right)}}\)

Check where denominator become zero

We know cos x = 0, where x = (2n + 1) π/2

So for cos 2x = 0 where, 2x = (2n + 1) π/2

x = (2n + 1) π/4

∴ Given function is discontinuous at x = (2n + 1) π/4

Hence, option (4) is correct.

Hint:

  • When dealing with a rational expression in which both the numerator and denominator are continuous.
  • The only points in which the rational expression will be discontinuous where denominator become zero.

\(\mathop {\lim }\limits_{{\rm{x}} \to 2} \sqrt {{{\rm{x}}^2} + 5} = ?\)

  1. √5
  2. √7
  3. √9
  4. Not possible

Answer (Detailed Solution Below)

Option 3 : √9

Concept:

  • \(\mathop {\lim }\limits_{{\rm{x}} \to {\rm{a}}} \sqrt[{\rm{n}}]{{{\rm{f}}\left( {\rm{x}} \right)}} = \sqrt[{\rm{n}}]{{\mathop {\lim }\limits_{{\rm{x}} \to {\rm{a}}} {\rm{f}}\left( {\rm{x}} \right)}}\)
  • \(\mathop {\lim }\limits_{{\rm{x}} \to {\rm{a}}} {\rm{f}}\left( {\rm{x}} \right) + {\rm{g}}\left( {\rm{x}} \right) = \mathop {{\rm{lim}}}\limits_{{\rm{x}} \to {\rm{a}}} {\rm{f}}\left( {\rm{x}} \right) + \mathop {{\rm{lim}}}\limits_{{\rm{x}} \to {\rm{a}}} {\rm{g}}\left( {\rm{x}} \right)\)

Calculation:

To find:\(\mathop {\lim }\limits_{{\rm{x}} \to 2} \sqrt {{{\rm{x}}^2} + 5} \) = ?

\(\mathop {\lim }\limits_{{\rm{x}} \to 2} \sqrt {{{\rm{x}}^2} + 5} = \sqrt {\mathop {\lim }\limits_{{\rm{x}} \to 2}( {{\rm{x}}^2} + 5}) \)

\(= \sqrt {\mathop {\lim }\limits_{{\rm{x}} \to 2} {{\rm{x}}^2} + \mathop {{\rm{lim}}}\limits_{{\rm{x}} \to 2} 5} = \sqrt {{2^2} + 5} \)

\(= \sqrt 9 \)

Hence, option (3) is correct.

Evaluate\(\mathop {\lim }\limits_{{\rm{x\;}} \to 0} \frac{{\rm \sqrt {a+x} - {\rm{\;}}\sqrt {\rm a - x} }}{\rm 4x}\)

  1. \( \frac{1}{{2\sqrt 2 }}\)
  2. \( \frac{1}{{\sqrt 2 }}\)
  3. \(\frac{1}{2}\)
  4. None of the above

Answer (Detailed Solution Below)

Option 4 : None of the above

Calculation:

We have to find the value of\(\mathop {\lim }\limits_{{\rm{x\;}} \to 0} \frac{{\rm \sqrt {a+x} - {\rm{\;}}\sqrt {\rm a - x} }}{\rm 4x}\)

Rationalise the numerator, we get

\(\mathop {\lim }\limits_{{\rm{x\;}} \to 0} \frac{{\rm \sqrt {a+x} - {\rm{\;}}\sqrt {\rm a - x} }}{\rm 4x} \times \frac{{\rm \sqrt {a+x} + {\rm{\;}}\sqrt {\rm a - x} }}{{\rm \sqrt {a+x} + {\rm{\;}}\sqrt {\rm a - x} }} \)

=\(\mathop {\lim }\limits_{{\rm{x\;}} \to 0} \frac{{\rm {a+x} - {\rm{\;}} {\rm (a - x )} }}{\rm 4x({\rm \sqrt {a+x} + {\rm{\;}}\sqrt {\rm a - x} } )} \)

=\(\mathop {\lim }\limits_{{\rm{x\;}} \to 0} \frac{\rm 2x}{\rm 4x({\rm \sqrt {a+x} + {\rm{\;}}\sqrt {\rm a - x} } )} \)

=\(\mathop {\lim }\limits_{{\rm{x\;}} \to 0} \frac{\rm 1}{\rm 2({\rm \sqrt {a+x} + {\rm{\;}}\sqrt {\rm a - x} } )} \)

=\(\frac{1}{{4 \rm \sqrt a }}\)

Find the value of\(\rm \displaystyle \lim_{n \rightarrow \infty} \frac{1-n^2} {\sum n}\)

  1. -2
  2. 2
  3. 1
  4. - 1

Answer (Detailed Solution Below)

Option 1 : -2

Concept:

\(\rm \sum n = \frac{n(n+1)}{2}\)

Calculation:

\(\rm \displaystyle \lim_{n \rightarrow \infty} \frac{1-n^2} {\sum n}\)

=\(\rm \displaystyle \lim_{n \rightarrow \infty} \frac{1-n^2} { \left[\frac{n(n+1)}{2} \right ]}\)

=\(\rm \displaystyle \lim_{n \rightarrow \infty} \frac{2(1-n^2)} {n^2+n}\)

Divide both numerator and denominator by n2

=\(\rm \displaystyle \lim_{n \rightarrow \infty} \frac{2\left[\frac{1-n^2}{n^2} \right ]} {\left[\frac{n^2+n}{n^2} \right ]}\)

=\(\rm \displaystyle \lim_{n \rightarrow \infty} \frac{2\left[\frac{1}{n^2} - 1 \right ]} {1 + \frac{1}{n} }\)

=\(\rm \frac{2(0-1)}{(1+0)}\)

= -2

hookerarriess.blogspot.com

Source: https://testbook.com/objective-questions/mcq-on-limit-and-continuity--5eea6a1439140f30f369f1a7